Nachweis der kanonischen Kommutierungsbeziehung (CCR)

Ich bin mir nicht sicher, wie Q P P Q = ich Wo P repräsentieren Schwung und Q Stellung darstellen. Q Und P sind Matrizen. Die Frage wäre, wie kann Q Und P als Matrix formuliert werden? Was ist auch der Beweis dieser kanonischen Kommutierungsrelation ?

Dies ist ein zentrales Axiom der Matrizenmechanik. Es kann in der Wellenmechanik demonstriert werden, aber dann fragen Sie: "Warum ist Pa ein Differentialoperator?". Die Herleitung ist heuristisch, sie funktioniert nur innerhalb der alten Quantentheorie und ist auf der Wikipedia-Seite "Matrixmechanik" wiedergegeben.
Interessanter Standpunkt zur kanonischen Kommutierungsrelation: johncostella.com/physics/…

Antworten (3)

Wie Lubos schon erwähnt hat

Q P P Q = ich

ist eine der Grundvoraussetzungen der Quantenmechanik. Klassische Observablen sind Funktionen von Variablen Q , Und P und Poisson-Klammer-Beziehung lesen

{ Q , P } = 1 (beachten Sie, dass { Q , P } ist einheitslose Größe )

In der QM müssen Observable hermitesche Operatoren sein (damit sie reelle Eigenwerte haben können). Insbesondere für die Position haben wir einen Operator Q , und für Impuls haben wir einen Operator P . Poisson-Klammer wird durch Kommutator ersetzt und wir benötigen

[ Q , P ] = ich

In Analogie zu klassischen Poisson-Klammern hätten wir benötigt

[ Q , P ] = 1

Aber das ist seither nicht möglich

i) Das haben wir bereits gefordert Q , P hermitesch sein. So [ Q , P ] = ( Q P P Q ) = ( Q P ) ( P Q ) = P Q Q P = [ Q , P ] . Also, wenn wir es brauchen [ Q , P ] Um eine Konstante zu sein (dh ein konstantes Vielfaches der Identitätsmatrix), sollte sie rein imaginär sein.

ii) [ Q , P ] hat Einheiten von M L 2 T 1 . Sie können dies sehen, indem Sie das notieren Q ist ein Positionsoperator, hat also Einheiten von L , und das P ist ein Impulsoperator, hat also Einheiten von M L T 1 .

Zwei natürliche Möglichkeiten sind [ Q , P ] = ich Und [ Q , P ] = ich . Sie sind sowohl gleichwertig als auch wählbar [ Q , P ] = ich ist nur eine Konvention.

Keine zwei endlichdimensionalen Matrizen können befriedigen [ Q , P ] = ich . Dies kann durch Aufzeichnen auf beiden Seiten gesehen werden. Diese Beziehung kann jedoch durch unendlich dimensionale Matrizen erfüllt werden. Nehmen Sie den Vektorraum expliziter als Raum von Funktionen von Q . Definieren Q als Q F = Q F , Und P als P F = ich F / Q . Dann ist ersichtlich, dass diese Operatoren die erforderliche Kommutierungsbeziehung erfüllen. Außerdem, wenn wir unser inneres Produkt definieren als

( F , G ) = F G D Q

Dann Q Und P wie oben definiert, wird auch hermitesch sein.

Nur zur Kontrolle: wie bekommen wir das hin [ Q , P ] hat Einheiten von M L 2 T 1 ?
@Krieg Q ist ein Positionsoperator, daher sind Längeneinheiten erforderlich L . Ähnlich P benötigt Impulseinheiten M L T 1 . So Q P P Q sollte Abmessungen von haben M L 2 T 1 .
@War Im letzten Absatz der obigen Antwort muss eine Korrektur vorgenommen werden. Definition von P sollte sein P F = ich F / Q .
@dushya, ich habe den Inhalt Ihrer Kommentare in die Antwort aufgenommen. Ich hoffe, dass das in Ordnung ist.

In einer Matrixdarstellung Q Und P sind unendlichdimensionale Matrizen. (Endlich dimensionale Matrizen würden nicht ausreichen; die Spur der beiden Seiten der Kommutierungsbeziehung ergibt einen Widerspruch.)

Viele mögliche Matrizenpaare qualifizieren sich; die schönsten erhält man, wenn man Ort und Impuls in einer Basis von Eigenzuständen des harmonischen Oszillators ausdrückt. Siehe
http://en.wikipedia.org/wiki/Matrix_mechanics#Harmonic_oscillator
Tatsächlich ist dies Heisenbergs ursprüngliche Darstellung für „Matrixmechanik“.)

Die häufiger verwendete Positionsdarstellung (oder Impulsdarstellung) nimmt Q (bzw. P ) als Multiplikationsoperator auf Wellenfunktionen in Abhängigkeit vom Ort (oder Impuls), und P (bzw. Q ) als Differentialoperator erster Ordnung, der passend zur Kommutierungsrelation gewählt wurde.

Um die Beziehung zu beweisen, wann Q Und P gegeben sind, wenden Sie einfach beide Seiten auf einen beliebigen Zustandsvektor an und überprüfen Sie, ob man dasselbe Ergebnis erhält.

Für φ eine glatte Schwartz-Funktion S ( R ) der einfachste Fall, den das CCR verallgemeinert, ist ( Q φ ) ( X ) = X φ ( X ) , ( P φ ) ( X ) = ich φ ' ( X ) , > 0 wir haben beide selbstadjungierte Operatoren ( P Q φ ) ( X ) = ich φ ( X ) ich X φ ' ( X ) , ( Q P φ ) ( X ) = ich X φ ' ( X ) So [ Q , P ] φ ( X ) = ( Q P P Q ) φ ( X ) = ich φ ( X ) .

Man kann allgemeinere Regeln formulieren, die die Kommutatoren von Operatoren in einer Quantentheorie bestimmen, die von einem Lagrange- oder einem Hamilton-Operator erhalten wird. Allerdings muss es immer einige Axiome geben. In den einfachsten Modellen der Quantenmechanik ist es am legitimsten, das zu sagen X P P X = ich ist einfach ein Schlüsselaxiom der Quantenmechanik, kann also nicht aus etwas "Tieferem" abgeleitet werden.

Die Tatsache, dass sie nicht pendeln – dass ihr Produkt von der Bestellung abhängt – impliziert dies X , P können keine gewöhnlichen Zahlen sein. Stattdessen sind sie Bediener: Bediener müssen nicht pendeln. Ein L ^ Operator ist etwas, das jedem Vektor zuweist | ψ eines Raumes – in der Quantenmechanik Hilbertraum – das Ergebnis L ^ | ψ .

Wählt man eine Basis des Raums mit endlich oder abzählbar vielen Elementen, so lassen sich alle Informationen über einen linearen Operator in Form von Matrixelementen ausdrücken ich | L ^ | J = B ( J , L ( ich ) ) und die Menge dieser inneren Produkte, dh Matrixelemente, ist eine Matrix.

Dieses Bild der Physik ist konsistent, aber die Konsistenz hat viele Aspekte, also ist es eine weit gefasste Frage. Im Wesentlichen wollen Sie erklären, warum alles in der Quantenmechanik funktioniert. Nun, das tut es, aber es ist kein 1-zeiliger Beweis. Die Quantenmechanik ist die richtige Theorie von allem, daher wäre es unklug, 1-zeilige Beweise für ihre Konsistenz oder Gültigkeit zu erwarten. Wenn Sie speziellere Bedenken oder hypothetische Übereinstimmungen haben, aktualisieren Sie bitte Ihre Frage.

Mehr kann nämlich gesagt werden, zumindest einer von X Und P muss ein unbeschränkter Operator auf dem Zustandsraum sein. Helmut Wielandt bewies als erster, dass kein Paar beschränkter Operatoren auf einem Hilbert-Raum jemals die Heisenberg-Kommutationsbeziehung erfüllen kann.